User avatar
 
bbirdwell
Thanks Received: 864
Atticus Finch
Atticus Finch
 
Posts: 803
Joined: April 16th, 2009
 
This post thanked 3 times.
 
 

Re: Q13 - Consumers seek to purchase

by bbirdwell Fri Dec 31, 1999 8:00 pm

The answers to these kinds of questions can be consistently predictable if your conditional logic is on the money. Once symbolized, you look at the conclusion and figure out which additional conditional statement is needed, in addition to the given premises, to guarantee the conclusion.

It's usually modeled something like this:
A--> B
C --> D
Conclusion:
A --> D

In order to do guarantee that A-->D, what can we add to the above evidence? What connection does the evidence cry out for? B-->C! That would complete the circuit in the most elementary way possible. On an easier problem, the answer will look something like B->C. On a more difficult problem, the answer will be the contrapositive, ~C-->~B.

I find it confusing sometimes to use only letters when I symbolize tough logical reasoning arguments, so I often use words or phrases.

Here's how I think about this one.

1st sentence: Hmm... The word "at" is kinda weird here. Is it an "and" or an "or"? Let's leave it aside for a second and look at the next one.

2nd sentence: This one is more clear.
~products that attract --> Bankrupt

Conclusion:
~Best quality & ~Lowest price -->Bankrupt

Hmm. The conclusion seems to be about two separate, absolute categories: BEST and lowEST. I don't think this really matches the first sentence, which is more of a spectrum of the two categories mixed together: best AT lowest.

I'm gonna ignore that for a moment and see if there's an easy way to go from the 2nd premise to the conclusion. The argument kinda looks like this now:
~C -->D
therefore: ~A and ~B --> D

Now, if I want to start and "~A and ~B" and end up at "D" using the one given piece of evidence, which puzzle piece do I need to add?

Well, starting at "~A and ~B," I can go through "~C" and that would lead directly to the desired "D." Let's try that and see if there's a good match. We'll go to the choices and look for either
~A and ~B --> ~C,
or the contrapositive, C --> A or B

Thinking in the argument's terms, if we know that "~highest quality & ~lowest price --> ~attracting consumers," we guarantee the conclusion, so this is likely to be the answer.

A) Highest quality --> ~Lowest price. Not what we need!
B) ~Highest quality and ~Lowest price --> ~attract. Perfect match!
C) Highest quality or Lowest price --> ~Bankrupt
D) not even close
E) not even close

Lemme know if that clears things up.


#officialexplanation
I host free online workshop/Q&A sessions called Zen and the Art of LSAT. You can find upcoming dates here: http://www.manhattanlsat.com/zen-and-the-art.cfm
 
peg_city
Thanks Received: 3
Forum Guests
 
Posts: 152
Joined: January 31st, 2011
Location: Winnipeg
 
 
trophy
First Responder
 

Q13 - Consumers seek to purchase

by peg_city Tue May 24, 2011 3:29 pm

I'm trying conditional logic for this one

C-> HQ + LP

~C->B

Conclusion:
~HQ or ~LP -> B

The answer is B, ~HQ or ~LP -> ~C

My question is, how am I supposed to know, without looking at the answers, that the contra-positive of the first inference will allow to conclusion to be properly drawn?

Thanks again
 
peg_city
Thanks Received: 3
Forum Guests
 
Posts: 152
Joined: January 31st, 2011
Location: Winnipeg
 
 
trophy
First Responder
 

Re: Q13 - Consumers seek to purchase

by peg_city Mon Aug 08, 2011 3:54 pm

I don't know why we would need to assume this if we can infer it from the conditional logic chain....

There is really no assumption required

what am I missing?
 
timmydoeslsat
Thanks Received: 887
Atticus Finch
Atticus Finch
 
Posts: 1136
Joined: June 20th, 2011
 
 
trophy
Most Thanked
trophy
First Responder
 

Re: Q13 - Consumers seek to purchase

by timmydoeslsat Mon Aug 08, 2011 5:12 pm

The correct answer for this question will be an assumption that is sufficient for the conclusion to be true, but it will not be required.

The conclusion of this argument is:

~Best Quality and ~ Lowest Price ---> Bankrupt

Evidence for this is:

Consumers seek to purchase the highest quality at the lowest prices. I am with Brian on this one, this is something I would set aside until I see that I need this. I do not see a conditional relationship in this statement.

So the other piece of evidence for this is something that is conditional and we will work with that:

~ Offer Products that attract customers ---> Bankrupt

Let us put these with one another to see this more clearly.



~ Products attract customers ---> Bankrupt
------------------------------------------------------
~Best Quality and ~ Lowest Price ---> Bankrupt

We can tell that more than likely, our answer choice will include the terms "best quality" and "lowest price" since this is in the conclusion and not in the premises as of now.

Choice B states something that we currently do not have:

~Best Quality and ~Lowest Price ---> ~Products attract customers

That would then, give us that newly formed premise along with our original premise. As you can see, we can validly conclude with that statement now.

~Best Quality and ~Lowest Price ---> ~Products attract customers
~ Products attract customers ---> Bankrupt
------------------------------------------------------
~Best Quality and ~ Lowest Price ---> Bankrupt
 
jennifer
Thanks Received: 0
Elle Woods
Elle Woods
 
Posts: 91
Joined: July 29th, 2010
 
 
 

Re: Q13 - Consumers seek to purchase

by jennifer Thu Nov 03, 2011 9:15 pm

I am having difficultly identifying the conditional logic, also I. A question such as this it takes me way too long. Can this question be answered without the use of CL. I selected answer choice C, but B is the correct answer. I eliminated the rest
User avatar
 
bbirdwell
Thanks Received: 864
Atticus Finch
Atticus Finch
 
Posts: 803
Joined: April 16th, 2009
 
 
 

Re: Q13 - Consumers seek to purchase

by bbirdwell Mon Nov 07, 2011 10:06 am

You don't necessarily have to diagram in order to understand the conditional logic, though many people, including myself, find it essential. As far as the "conditional logic" itself, many people find that to be a difficult aspect of the test, so you are definitely not alone.

My advice would be to practice, practice, practice until you are better at it because conditional logic is the beating heart of the entire test; in fact, it is the same current of thinking that lies beneath written law, and that is why it's an indispensable skill for attending law school.
I host free online workshop/Q&A sessions called Zen and the Art of LSAT. You can find upcoming dates here: http://www.manhattanlsat.com/zen-and-the-art.cfm
 
karynomisanya
Thanks Received: 0
Vinny Gambini
Vinny Gambini
 
Posts: 1
Joined: March 01st, 2014
 
 
 

Re: Q13 - Consumers seek to purchase

by karynomisanya Sat Nov 12, 2016 10:34 pm

Can someone go through answer choice "C" and explain why it's wrong.
 
lire.moi
Thanks Received: 0
Vinny Gambini
Vinny Gambini
 
Posts: 3
Joined: February 17th, 2017
 
 
 

Re: Q13 - Consumers seek to purchase

by lire.moi Tue Mar 07, 2017 10:01 pm

I too felt like C worked, but on second look I think I’ve figured out our problem.

Looking at answer choice C again:

If a company offers either HQ or LP -> Avoids Bankruptcy

Let’s think about what happens to that conditional if I have a company that offers neither HQ nor LP….. I need to negate both elements in the sufficient condition. BUT that doesn’t tell me anything about whether I avoided bankruptcy or not! Assuming that outcome is a mistaken negation/not a valid inference. All we know FOR SURE from answer choice C is that if a company offers either HQ or LP, they won’t go bankrupt. But based on this answer it is still possible that a company offers neither and STILL find a way to avoid bankruptcy. Just because you are no longer guaranteed to avoid bankruptcy doesn’t mean you MUST have gone bankrupt.

Answer choice B on the other hand guarantees that if I don’t offer HQ and LP, I won’t attract customers, which in turn guarantees that I WILL go bankrupt. (Similarly in this answer choice there is the possibility to offer either HQ or LP or both and still go bankrupt. Maybe you offered HQ and LP but your brother-in-law embezzled all the money.) Since we are looking for a connection that forces a company to go bankrupt, this answer choice works.

The point is that the argument in the stimulus doesn’t make any claim about what guarantees avoiding bankruptcy. So we don’t care about any answer choice that deals with that. We only care about answer choices that help us understand what guarantees bankruptcy. And B gives us the proper connection.
 
HelenH783
Thanks Received: 1
Vinny Gambini
Vinny Gambini
 
Posts: 11
Joined: October 26th, 2017
 
 
 

Re: Q13 - Consumers seek to purchase

by HelenH783 Mon Nov 06, 2017 10:18 pm

I get thrown off by the fact that the first sentence in the prompt, "Consumers seek to purchase the highest quality at the lowest prices," seems like the contrapositive of the correct answer choice, "Products that are neither highest in quality nor lowest in price do not attract customers." In this way, I read the answer choice as already being included as a premise. I understand the conditional logic mapping route to getting the right answer, but why can we ignore the first sentence? It seems to me that the argument already logically follows.
 
Lisaandpeterlin
Thanks Received: 2
Vinny Gambini
Vinny Gambini
 
Posts: 6
Joined: July 10th, 2016
 
 
 

Re: Q13 - Consumers seek to purchase

by Lisaandpeterlin Fri Jul 13, 2018 9:57 am

bbirdwell Wrote:The answers to these kinds of questions can be consistently predictable if your conditional logic is on the money. Once symbolized, you look at the conclusion and figure out which additional conditional statement is needed, in addition to the given premises, to guarantee the conclusion.

It's usually modeled something like this:
A--> B
C --> D
Conclusion:
A --> D

In order to do guarantee that A-->D, what can we add to the above evidence? What connection does the evidence cry out for? B-->C! That would complete the circuit in the most elementary way possible. On an easier problem, the answer will look something like B->C. On a more difficult problem, the answer will be the contrapositive, ~C-->~B.

I find it confusing sometimes to use only letters when I symbolize tough logical reasoning arguments, so I often use words or phrases.

Here's how I think about this one.

1st sentence: Hmm... The word "at" is kinda weird here. Is it an "and" or an "or"? Let's leave it aside for a second and look at the next one.

2nd sentence: This one is more clear.
~products that attract --> Bankrupt

Conclusion:
~Best quality & ~Lowest price -->Bankrupt

Hmm. The conclusion seems to be about two separate, absolute categories: BEST and lowEST. I don't think this really matches the first sentence, which is more of a spectrum of the two categories mixed together: best AT lowest.

I'm gonna ignore that for a moment and see if there's an easy way to go from the 2nd premise to the conclusion. The argument kinda looks like this now:
~C -->D
therefore: ~A and ~B --> D

Now, if I want to start and "~A and ~B" and end up at "D" using the one given piece of evidence, which puzzle piece do I need to add?

Well, starting at "~A and ~B," I can go through "~C" and that would lead directly to the desired "D." Let's try that and see if there's a good match. We'll go to the choices and look for either
~A and ~B --> ~C,
or the contrapositive, C --> A or B

Thinking in the argument's terms, if we know that "~highest quality & ~lowest price --> ~attracting consumers," we guarantee the conclusion, so this is likely to be the answer.

A) Highest quality --> ~Lowest price. Not what we need!
B) ~Highest quality and ~Lowest price --> ~attract. Perfect match!
C) Highest quality or Lowest price --> ~Bankrupt
D) not even close
E) not even close

Lemme know if that clears things up.


#officialexplanation



So - to clarify - when using the neither/nor - its "or"? I'm confused - I thought on analytical reasoning (if I remember correctly that means "and"). I guess that's why I chose B verses A is because of the and/or rule.
 
Yit HanS103
Thanks Received: 2
Vinny Gambini
Vinny Gambini
 
Posts: 24
Joined: November 07th, 2017
 
This post thanked 1 time.
 
 

Re: Q13 - Consumers seek to purchase

by Yit HanS103 Tue Sep 04, 2018 8:40 pm

I need help here please!!

B to me it sounds like a premise booster.
the premise: "companies that do not offer PRODUCTS that attract consumers ..." Sounds like a repetition of b. I didn't like any of the 5 choices but under time I chose E.
E defends the fact that there aren't other reasons besides meeting consumer demands. Therefore bankrupt. though, I wasn't really sure about E either.


Thank you!!
 
EaronR591
Thanks Received: 1
Vinny Gambini
Vinny Gambini
 
Posts: 3
Joined: September 08th, 2018
 
This post thanked 1 time.
 
 

Re: Q13 - Consumers seek to purchase

by EaronR591 Tue Oct 02, 2018 11:24 am

Yit HanS103 Wrote:I need help here please!!

B to me it sounds like a premise booster.
the premise: "companies that do not offer PRODUCTS that attract consumers ..." Sounds like a repetition of b. I didn't like any of the 5 choices but under time I chose E.
E defends the fact that there aren't other reasons besides meeting consumer demands. Therefore bankrupt. though, I wasn't really sure about E either.


Thank you!!


B is not a Premise Booster because it makes a connection that did not previously exist: connecting highest quality and/or lowest prices to the attraction of customers to companies. The second sentences only uses elements of companies not having products which attract consumers -> bankruptcy. There is nothing in that premise which refers to highest quality and/or lowest price. Without the assumption of B, the premise of sentence two allows for other reasons for not attracting consumers -> bankruptcy. Maybe the store only sells peanuts and is located in a town of only people with nut allergies....that store wouldn't attract consumers as well, and therefore go bankrupt.

In THIS case, however, the conclusion suggests that offering neither best quality nor lowest price causes bankruptcy...we have nothing that connects that. We know that not attracting consumers causes bankruptcy, not anything about quality/price. That connection needs to be added:

Dont Attract Consumers -> Bankrupt
Not Highest Quality/Lowest Price -> Bankrupt
Need to add Not Highest Quality/Lowest Price -> Dont Attract Consumers -> Bankrupt
Now that the chain is longer, we can remove the middle piece (Dont Attract Consumers) and go directly from Not Highest Quality/Low Price -> Bankrupt

Note: The first sentence, I believe, is intended to confuse, because it talk about the desire consumers for highest quality and lowest prices. It only talks about them wanting to purchase...not them being attracted to products. Note sentence 1 does not mention Products nor the idea of being attracted as a consumer. I think this is a tough sentence because it seems like an easy logical jump to think: "Hey, consumer seeks to purchase highest quality @ lowest prices, then they most be attracted to companies that offer highest quality and/or lowest price." While it makes sense in our heads, we are adding a piece that does not exist.....answer B spells out what our brains probably did automatically.

I hope that helped??